Quiz-summary
0 of 30 questions completed
Questions:
- 1
- 2
- 3
- 4
- 5
- 6
- 7
- 8
- 9
- 10
- 11
- 12
- 13
- 14
- 15
- 16
- 17
- 18
- 19
- 20
- 21
- 22
- 23
- 24
- 25
- 26
- 27
- 28
- 29
- 30
Information
Premium Practice Questions
You have already completed the quiz before. Hence you can not start it again.
Quiz is loading...
You must sign in or sign up to start the quiz.
You have to finish following quiz, to start this quiz:
Results
0 of 30 questions answered correctly
Your time:
Time has elapsed
Categories
- Not categorized 0%
- 1
- 2
- 3
- 4
- 5
- 6
- 7
- 8
- 9
- 10
- 11
- 12
- 13
- 14
- 15
- 16
- 17
- 18
- 19
- 20
- 21
- 22
- 23
- 24
- 25
- 26
- 27
- 28
- 29
- 30
- Answered
- Review
-
Question 1 of 30
1. Question
A 70-year-old patient is admitted to the orthopedic unit following a hip fracture. The patient is a devout Jehovah’s Witness and refuses blood transfusions due to religious beliefs. The patient’s hemoglobin level is 7.0 g/dL, and the physician recommends a blood transfusion to improve oxygen-carrying capacity and facilitate healing. The patient remains adamant in their refusal, despite the physician’s explanation of the risks associated with anemia. What is the MOST ethically and legally appropriate nursing action in this situation?
Correct
The scenario presents a complex ethical dilemma involving patient autonomy, cultural beliefs, and the potential for harm. The patient’s refusal of a blood transfusion, based on religious grounds, must be respected to the extent possible, but the nurse also has a duty to prevent harm and preserve life. Obtaining a court order to override the patient’s refusal is a drastic measure that should only be considered as a last resort when all other options have been exhausted. Discharging the patient against medical advice without addressing the underlying anemia is unethical and could lead to serious harm or death. Encouraging the patient to reconsider their decision is appropriate, but it must be done in a non-coercive manner, respecting their autonomy and cultural beliefs. The most appropriate action is to collaborate with the healthcare team, including the physician, social worker, and potentially a religious advisor, to explore alternative treatment options that are acceptable to the patient and address the anemia. This may involve using erythropoietin-stimulating agents, iron supplementation, or other blood-conserving strategies. It is also important to provide the patient with comprehensive information about the risks and benefits of each option, allowing them to make an informed decision that aligns with their values and beliefs.
Incorrect
The scenario presents a complex ethical dilemma involving patient autonomy, cultural beliefs, and the potential for harm. The patient’s refusal of a blood transfusion, based on religious grounds, must be respected to the extent possible, but the nurse also has a duty to prevent harm and preserve life. Obtaining a court order to override the patient’s refusal is a drastic measure that should only be considered as a last resort when all other options have been exhausted. Discharging the patient against medical advice without addressing the underlying anemia is unethical and could lead to serious harm or death. Encouraging the patient to reconsider their decision is appropriate, but it must be done in a non-coercive manner, respecting their autonomy and cultural beliefs. The most appropriate action is to collaborate with the healthcare team, including the physician, social worker, and potentially a religious advisor, to explore alternative treatment options that are acceptable to the patient and address the anemia. This may involve using erythropoietin-stimulating agents, iron supplementation, or other blood-conserving strategies. It is also important to provide the patient with comprehensive information about the risks and benefits of each option, allowing them to make an informed decision that aligns with their values and beliefs.
-
Question 2 of 30
2. Question
An 82-year-old female with a history of osteoporosis, chronic lower back pain managed with long-term opioid therapy, peripheral neuropathy secondary to diabetes, and a 10-year history of bisphosphonate use presents to the emergency department after a fall. Radiographs reveal a mid-shaft femur fracture. She reports severe pain, which she rates as 9/10 despite receiving her usual dose of oral opioid medication at home. Her serum vitamin D level is 18 ng/mL (normal range: 30-100 ng/mL). Which of the following nursing interventions is the *most* comprehensive and appropriate initial approach to managing this patient’s care, considering her complex medical history and the new fracture?
Correct
The scenario presents a patient with a complex orthopedic history and a new fracture. The nurse must consider the patient’s pre-existing conditions, medications, and the potential impact of these factors on fracture healing and pain management. Bisphosphonates, while beneficial for osteoporosis, can, in rare instances, be associated with atypical femoral fractures and might influence bone healing. Chronic opioid use can lead to opioid-induced hyperalgesia, where the patient becomes more sensitive to pain, making pain management challenging. Vitamin D deficiency, common in older adults, impairs calcium absorption, which is crucial for bone healing. Finally, peripheral neuropathy can complicate pain assessment and management, as the patient may experience altered pain sensations. Therefore, the most comprehensive approach involves addressing all these factors: assessing the potential impact of bisphosphonates on fracture healing (though not necessarily discontinuing them immediately without physician order), recognizing the possibility of opioid-induced hyperalgesia, optimizing vitamin D levels, and carefully evaluating the patient’s pain in the context of their peripheral neuropathy. A multidisciplinary approach involving the orthopedic surgeon, pain management specialist, and physical therapist is essential. Focusing solely on one aspect, such as opioid dosage or vitamin D supplementation, would be insufficient in managing this complex patient.
Incorrect
The scenario presents a patient with a complex orthopedic history and a new fracture. The nurse must consider the patient’s pre-existing conditions, medications, and the potential impact of these factors on fracture healing and pain management. Bisphosphonates, while beneficial for osteoporosis, can, in rare instances, be associated with atypical femoral fractures and might influence bone healing. Chronic opioid use can lead to opioid-induced hyperalgesia, where the patient becomes more sensitive to pain, making pain management challenging. Vitamin D deficiency, common in older adults, impairs calcium absorption, which is crucial for bone healing. Finally, peripheral neuropathy can complicate pain assessment and management, as the patient may experience altered pain sensations. Therefore, the most comprehensive approach involves addressing all these factors: assessing the potential impact of bisphosphonates on fracture healing (though not necessarily discontinuing them immediately without physician order), recognizing the possibility of opioid-induced hyperalgesia, optimizing vitamin D levels, and carefully evaluating the patient’s pain in the context of their peripheral neuropathy. A multidisciplinary approach involving the orthopedic surgeon, pain management specialist, and physical therapist is essential. Focusing solely on one aspect, such as opioid dosage or vitamin D supplementation, would be insufficient in managing this complex patient.
-
Question 3 of 30
3. Question
A 25-year-old male patient sustained a tibial fracture in a motorcycle accident and underwent open reduction and internal fixation (ORIF). The patient reports severe pain in his lower leg that is unrelieved by opioid analgesics. Which of the following is the MOST reliable early indicator of compartment syndrome in this patient?
Correct
Compartment syndrome is a condition in which increased pressure within a confined anatomical space compromises the blood supply to tissues within that space. This can lead to muscle and nerve damage, potentially resulting in permanent disability if not treated promptly. The “6 Ps” are classic signs and symptoms of compartment syndrome: pain (out of proportion to the injury), pallor (paleness), paresthesia (numbness or tingling), paralysis (weakness or inability to move), pulselessness (diminished or absent pulse), and poikilothermia (coolness). However, pulselessness is a late sign and may not always be present, especially in the early stages of compartment syndrome. The earliest and most reliable sign is often pain that is out of proportion to the injury and is not relieved by pain medication. Paresthesia, particularly numbness or tingling in the web space between the first and second toes (in the case of lower leg compartment syndrome), is also an early and important indicator. Pallor and poikilothermia may also be present, but they are less specific. Paralysis and pulselessness are late signs that indicate significant tissue damage and require immediate intervention. Therefore, the MOST reliable early indicator is pain out of proportion to the injury, coupled with paresthesia.
Incorrect
Compartment syndrome is a condition in which increased pressure within a confined anatomical space compromises the blood supply to tissues within that space. This can lead to muscle and nerve damage, potentially resulting in permanent disability if not treated promptly. The “6 Ps” are classic signs and symptoms of compartment syndrome: pain (out of proportion to the injury), pallor (paleness), paresthesia (numbness or tingling), paralysis (weakness or inability to move), pulselessness (diminished or absent pulse), and poikilothermia (coolness). However, pulselessness is a late sign and may not always be present, especially in the early stages of compartment syndrome. The earliest and most reliable sign is often pain that is out of proportion to the injury and is not relieved by pain medication. Paresthesia, particularly numbness or tingling in the web space between the first and second toes (in the case of lower leg compartment syndrome), is also an early and important indicator. Pallor and poikilothermia may also be present, but they are less specific. Paralysis and pulselessness are late signs that indicate significant tissue damage and require immediate intervention. Therefore, the MOST reliable early indicator is pain out of proportion to the injury, coupled with paresthesia.
-
Question 4 of 30
4. Question
A 62-year-old female is scheduled for an elective total hip arthroplasty. The surgeon has explained the procedure to the patient, and she has signed the consent form. Prior to administering the pre-operative medications, what is the MOST important action for the orthopedic nurse to take regarding the informed consent process, considering the ethical and legal responsibilities of the nurse in ensuring patient autonomy and understanding? The nurse understands that informed consent is an ongoing process, not just a one-time event. The nurse also recognizes the importance of patient education and advocacy.
Correct
The correct answer is to assess the patient’s understanding of the procedure, potential risks and benefits, and alternative treatment options. Obtaining informed consent is not merely about getting a signature on a form; it is about ensuring that the patient has the necessary information to make an autonomous decision about their healthcare. While the surgeon is responsible for explaining the surgical procedure itself, the nurse plays a crucial role in verifying the patient’s understanding. This includes assessing their comprehension of the risks and benefits, discussing alternative treatment options, and answering any questions they may have. Simply witnessing the signature does not guarantee that the patient is truly informed. Assuming the patient understands because they signed the form is a negligent practice. Asking the patient if they have any questions is a good starting point, but it does not actively assess their understanding. The nurse’s role is to be an advocate for the patient and to ensure that they are making a truly informed decision.
Incorrect
The correct answer is to assess the patient’s understanding of the procedure, potential risks and benefits, and alternative treatment options. Obtaining informed consent is not merely about getting a signature on a form; it is about ensuring that the patient has the necessary information to make an autonomous decision about their healthcare. While the surgeon is responsible for explaining the surgical procedure itself, the nurse plays a crucial role in verifying the patient’s understanding. This includes assessing their comprehension of the risks and benefits, discussing alternative treatment options, and answering any questions they may have. Simply witnessing the signature does not guarantee that the patient is truly informed. Assuming the patient understands because they signed the form is a negligent practice. Asking the patient if they have any questions is a good starting point, but it does not actively assess their understanding. The nurse’s role is to be an advocate for the patient and to ensure that they are making a truly informed decision.
-
Question 5 of 30
5. Question
A 68-year-old patient presents to the emergency department after a fall, sustaining a closed tibial plateau fracture. Initial assessment reveals significant pain, swelling, and limited range of motion in the affected leg. The patient reports that the pain is much worse than they would expect from the injury. Distal pulses are present, but weak. Capillary refill is slightly delayed. The patient winces with severe pain when the nurse passively dorsiflexes their foot. Which of the following nursing interventions is the MOST critical and should be implemented FIRST?
Correct
The scenario presents a patient with a tibial plateau fracture, a complex injury involving both bone and soft tissue damage. The key is to prioritize interventions based on the immediate threats to limb viability and long-term function. While pain management, wound care, and DVT prophylaxis are all important, the most critical initial action is assessing for and managing compartment syndrome. Compartment syndrome is a condition where increased pressure within a confined space (in this case, the leg compartments) compromises blood flow and nerve function. It can lead to irreversible muscle and nerve damage, potentially resulting in amputation if not addressed promptly. The classic “5 Ps” of compartment syndrome (pain, pallor, paresthesia, paralysis, pulselessness) are important to assess, but pain out of proportion to the injury, especially with passive stretching of the muscles in the affected compartment, is an early and reliable indicator. Prompt recognition requires frequent neurovascular assessments and, if suspected, immediate communication with the orthopedic surgeon for possible fasciotomy (surgical release of the pressure). While the other options are important aspects of care, they do not address the immediate threat to limb viability posed by compartment syndrome. Delaying intervention for compartment syndrome while focusing on other aspects of care could have devastating consequences for the patient. Effective pain management is crucial, but it should not overshadow the need to rule out and address compartment syndrome. Similarly, wound care and DVT prophylaxis are essential but are secondary to ensuring adequate tissue perfusion.
Incorrect
The scenario presents a patient with a tibial plateau fracture, a complex injury involving both bone and soft tissue damage. The key is to prioritize interventions based on the immediate threats to limb viability and long-term function. While pain management, wound care, and DVT prophylaxis are all important, the most critical initial action is assessing for and managing compartment syndrome. Compartment syndrome is a condition where increased pressure within a confined space (in this case, the leg compartments) compromises blood flow and nerve function. It can lead to irreversible muscle and nerve damage, potentially resulting in amputation if not addressed promptly. The classic “5 Ps” of compartment syndrome (pain, pallor, paresthesia, paralysis, pulselessness) are important to assess, but pain out of proportion to the injury, especially with passive stretching of the muscles in the affected compartment, is an early and reliable indicator. Prompt recognition requires frequent neurovascular assessments and, if suspected, immediate communication with the orthopedic surgeon for possible fasciotomy (surgical release of the pressure). While the other options are important aspects of care, they do not address the immediate threat to limb viability posed by compartment syndrome. Delaying intervention for compartment syndrome while focusing on other aspects of care could have devastating consequences for the patient. Effective pain management is crucial, but it should not overshadow the need to rule out and address compartment syndrome. Similarly, wound care and DVT prophylaxis are essential but are secondary to ensuring adequate tissue perfusion.
-
Question 6 of 30
6. Question
A 72-year-old female patient with a history of osteoporosis presents to the orthopedic clinic complaining of persistent, activity-related thigh pain for the past several weeks. She has been taking alendronate 70mg weekly for the past 8 years. Radiographs reveal a subtle, transverse fracture line in the subtrochanteric region of her left femur, consistent with an atypical femur fracture (AFF). Her serum 25-hydroxyvitamin D level is 28 ng/mL. Which of the following is the MOST appropriate initial management strategy for this patient, considering the risk of complete fracture and the need to address the underlying pathophysiology of AFFs? The patient is otherwise healthy and reports good adherence to her medication regimen. She denies any recent trauma or falls. She is able to ambulate with a cane, but reports increasing difficulty with activities of daily living due to the thigh pain. Her bone mineral density (BMD) T-score at the hip is -2.8.
Correct
The correct approach involves understanding the interplay between bisphosphonates, bone remodeling, and atypical femur fractures (AFFs). Bisphosphonates, while effective in increasing bone mineral density (BMD) and reducing the risk of osteoporotic fractures, can suppress bone turnover. This suppression, particularly with long-term use, can lead to the accumulation of microdamage in the femoral cortex. The suppressed bone remodeling prevents the repair of these microfractures, eventually leading to stress fractures that can propagate into AFFs. The atypical fractures are often transverse or short oblique in nature, occurring in the subtrochanteric or diaphyseal regions of the femur. Discontinuing bisphosphonates can allow bone remodeling to resume, potentially facilitating the healing of microdamage and reducing the risk of further AFFs. However, the suppressed bone turnover persists for a variable period after cessation, depending on the bisphosphonate’s half-life and duration of use. Therefore, discontinuing the medication is the first step, but additional interventions are crucial. Vitamin D supplementation is important to optimize bone health and facilitate bone remodeling once bisphosphonate suppression is reduced. Weight-bearing should be limited to prevent further stress on the affected femur. Prophylactic fixation with an intramedullary nail is often considered to stabilize the femur and prevent complete fracture, particularly if prodromal symptoms (e.g., thigh pain) are present or if imaging shows signs of an impending fracture. Monitoring BMD is essential to assess the ongoing risk of osteoporotic fractures and guide further management decisions. The decision to initiate alternative osteoporosis treatments (e.g., teriparatide, denosumab) must be carefully considered based on the patient’s overall fracture risk, BMD, and individual circumstances.
Incorrect
The correct approach involves understanding the interplay between bisphosphonates, bone remodeling, and atypical femur fractures (AFFs). Bisphosphonates, while effective in increasing bone mineral density (BMD) and reducing the risk of osteoporotic fractures, can suppress bone turnover. This suppression, particularly with long-term use, can lead to the accumulation of microdamage in the femoral cortex. The suppressed bone remodeling prevents the repair of these microfractures, eventually leading to stress fractures that can propagate into AFFs. The atypical fractures are often transverse or short oblique in nature, occurring in the subtrochanteric or diaphyseal regions of the femur. Discontinuing bisphosphonates can allow bone remodeling to resume, potentially facilitating the healing of microdamage and reducing the risk of further AFFs. However, the suppressed bone turnover persists for a variable period after cessation, depending on the bisphosphonate’s half-life and duration of use. Therefore, discontinuing the medication is the first step, but additional interventions are crucial. Vitamin D supplementation is important to optimize bone health and facilitate bone remodeling once bisphosphonate suppression is reduced. Weight-bearing should be limited to prevent further stress on the affected femur. Prophylactic fixation with an intramedullary nail is often considered to stabilize the femur and prevent complete fracture, particularly if prodromal symptoms (e.g., thigh pain) are present or if imaging shows signs of an impending fracture. Monitoring BMD is essential to assess the ongoing risk of osteoporotic fractures and guide further management decisions. The decision to initiate alternative osteoporosis treatments (e.g., teriparatide, denosumab) must be carefully considered based on the patient’s overall fracture risk, BMD, and individual circumstances.
-
Question 7 of 30
7. Question
A 40-year-old male with a spinal cord injury at the T6 level suddenly develops a severe headache, flushing, and nasal congestion. His blood pressure is 210/110 mmHg. Which of the following nursing interventions should be implemented *first*? The patient is alert and oriented, but anxious.
Correct
The scenario involves a patient with a spinal cord injury at the T6 level. Patients with spinal cord injuries at or above the T6 level are at risk for autonomic dysreflexia, a potentially life-threatening condition characterized by a sudden, massive discharge of the sympathetic nervous system in response to noxious stimuli below the level of the injury. Common triggers for autonomic dysreflexia include bladder distention, bowel impaction, skin irritation, and restrictive clothing. The *most* immediate nursing intervention is to elevate the head of the bed to at least 45 degrees to lower blood pressure. Then, the nurse should assess for and remove the triggering stimulus. Checking blood pressure is important to confirm the diagnosis, but elevating the head of the bed takes priority to reduce the risk of stroke or other complications. Administering oxygen may be necessary if the patient is hypoxic, but is not the initial intervention. Notifying the physician is important, but should be done after the initial interventions to address the immediate threat to the patient’s safety.
Incorrect
The scenario involves a patient with a spinal cord injury at the T6 level. Patients with spinal cord injuries at or above the T6 level are at risk for autonomic dysreflexia, a potentially life-threatening condition characterized by a sudden, massive discharge of the sympathetic nervous system in response to noxious stimuli below the level of the injury. Common triggers for autonomic dysreflexia include bladder distention, bowel impaction, skin irritation, and restrictive clothing. The *most* immediate nursing intervention is to elevate the head of the bed to at least 45 degrees to lower blood pressure. Then, the nurse should assess for and remove the triggering stimulus. Checking blood pressure is important to confirm the diagnosis, but elevating the head of the bed takes priority to reduce the risk of stroke or other complications. Administering oxygen may be necessary if the patient is hypoxic, but is not the initial intervention. Notifying the physician is important, but should be done after the initial interventions to address the immediate threat to the patient’s safety.
-
Question 8 of 30
8. Question
A 62-year-old male patient with a long-standing history of osteoarthritis (OA) presents to the clinic with acute onset of severe pain, redness, and swelling in his right great toe. The joint is hot to the touch and exquisitely tender to palpation. He reports that he has never experienced this level of pain before. The physician suspects a gout flare and prescribes colchicine. Which of the following nursing interventions is MOST appropriate?
Correct
This scenario presents a patient with a history of chronic osteoarthritis (OA) who is now experiencing acute symptoms of a gout flare. Gout is a form of inflammatory arthritis caused by the deposition of uric acid crystals in the joints. While OA and gout can coexist, the acute presentation of a hot, swollen, and exquisitely tender joint, particularly in the great toe (podagra), is highly suggestive of a gout flare. Applying heat might exacerbate the inflammation. Encouraging weight-bearing would be extremely painful and potentially harmful. NSAIDs are commonly used for both OA and gout, but colchicine is a specific medication used to treat and prevent gout flares. It works by reducing the inflammation caused by uric acid crystals. Therefore, the most appropriate action is to administer the prescribed colchicine to address the acute gout flare.
Incorrect
This scenario presents a patient with a history of chronic osteoarthritis (OA) who is now experiencing acute symptoms of a gout flare. Gout is a form of inflammatory arthritis caused by the deposition of uric acid crystals in the joints. While OA and gout can coexist, the acute presentation of a hot, swollen, and exquisitely tender joint, particularly in the great toe (podagra), is highly suggestive of a gout flare. Applying heat might exacerbate the inflammation. Encouraging weight-bearing would be extremely painful and potentially harmful. NSAIDs are commonly used for both OA and gout, but colchicine is a specific medication used to treat and prevent gout flares. It works by reducing the inflammation caused by uric acid crystals. Therefore, the most appropriate action is to administer the prescribed colchicine to address the acute gout flare.
-
Question 9 of 30
9. Question
A 78-year-old male patient is being transferred from the operating room to the medical-surgical unit following a hip fracture repair. The patient has a Jackson-Pratt (JP) drain in place at the surgical site. Which of the following nursing interventions is MOST appropriate for managing the JP drain?
Correct
This scenario presents a patient with a hip fracture who is being transferred from the operating room to the medical-surgical unit. The patient has a Jackson-Pratt (JP) drain in place. A JP drain is a closed-suction drain used to remove fluid from the surgical site. The purpose of the JP drain is to prevent hematoma formation and reduce the risk of infection. The drain should be compressed to create suction and facilitate drainage. The nurse should also monitor the amount and characteristics of the drainage. The drain should be emptied when it is half full or as ordered by the physician. The drain should be placed below the level of the hip to promote drainage. The drain should not be clamped unless specifically ordered by the physician.
Incorrect
This scenario presents a patient with a hip fracture who is being transferred from the operating room to the medical-surgical unit. The patient has a Jackson-Pratt (JP) drain in place. A JP drain is a closed-suction drain used to remove fluid from the surgical site. The purpose of the JP drain is to prevent hematoma formation and reduce the risk of infection. The drain should be compressed to create suction and facilitate drainage. The nurse should also monitor the amount and characteristics of the drainage. The drain should be emptied when it is half full or as ordered by the physician. The drain should be placed below the level of the hip to promote drainage. The drain should not be clamped unless specifically ordered by the physician.
-
Question 10 of 30
10. Question
An 85-year-old female with moderate cognitive impairment sustains a hip fracture after a fall. The orthopedic surgeon recommends surgical repair. The patient consistently refuses the surgery, stating she “just wants to be comfortable.” Her son insists that the surgery is necessary for her well-being and asks the nurse to convince his mother to consent. Which of the following nursing actions is MOST ethically appropriate?
Correct
This scenario focuses on the ethical principle of autonomy in the context of orthopedic care. Autonomy refers to the patient’s right to make informed decisions about their own medical treatment, free from coercion or undue influence. In this case, the patient, despite being elderly and having cognitive deficits, is still capable of expressing her wishes. Her consistent refusal of the surgery, even when presented with the potential benefits, demonstrates her exercise of autonomy. While the son’s desire to proceed with the surgery may stem from good intentions, overriding the patient’s expressed wishes would violate her autonomy. The nurse’s role is to advocate for the patient’s rights and ensure that her wishes are respected. Obtaining a psychiatric evaluation may be helpful in assessing the patient’s decision-making capacity, but it should not be used to circumvent her autonomy. Proceeding with surgery against the patient’s will or seeking a court order would be ethically inappropriate.
Incorrect
This scenario focuses on the ethical principle of autonomy in the context of orthopedic care. Autonomy refers to the patient’s right to make informed decisions about their own medical treatment, free from coercion or undue influence. In this case, the patient, despite being elderly and having cognitive deficits, is still capable of expressing her wishes. Her consistent refusal of the surgery, even when presented with the potential benefits, demonstrates her exercise of autonomy. While the son’s desire to proceed with the surgery may stem from good intentions, overriding the patient’s expressed wishes would violate her autonomy. The nurse’s role is to advocate for the patient’s rights and ensure that her wishes are respected. Obtaining a psychiatric evaluation may be helpful in assessing the patient’s decision-making capacity, but it should not be used to circumvent her autonomy. Proceeding with surgery against the patient’s will or seeking a court order would be ethically inappropriate.
-
Question 11 of 30
11. Question
A 25-year-old male presents to the emergency department with a tibial fracture sustained during a motorcycle accident. He underwent open reduction and internal fixation (ORIF) yesterday. The patient reports increasing pain in his lower leg that is not relieved by prescribed pain medication. On examination, the nurse notes significant swelling, tightness of the calf muscle, and paresthesia in the foot. Distal pulses are palpable, and capillary refill is within normal limits. Which of the following nursing interventions is the MOST appropriate initial action to address the patient’s clinical presentation? Consider the potential complications associated with fractures and the importance of timely intervention.
Correct
The scenario describes a patient experiencing increasing pain, swelling, and paresthesia in the lower leg following a tibial fracture. These are classic signs and symptoms of compartment syndrome, a condition where increased pressure within a confined muscle compartment compromises blood flow and nerve function. The “5 Ps” of compartment syndrome are pain (out of proportion to the injury), pallor, paresthesia, pulselessness, and paralysis. However, pulselessness is a late sign. The priority nursing intervention is to notify the physician immediately. Early recognition and prompt intervention are crucial to prevent irreversible muscle and nerve damage. Elevating the extremity can worsen compartment syndrome by decreasing arterial blood flow. Applying ice can also constrict blood vessels and further compromise circulation. Administering pain medication is important, but it does not address the underlying problem of increased compartment pressure. Therefore, the most appropriate initial action is to notify the physician immediately to facilitate prompt diagnosis and potential surgical intervention (fasciotomy) to relieve the pressure.
Incorrect
The scenario describes a patient experiencing increasing pain, swelling, and paresthesia in the lower leg following a tibial fracture. These are classic signs and symptoms of compartment syndrome, a condition where increased pressure within a confined muscle compartment compromises blood flow and nerve function. The “5 Ps” of compartment syndrome are pain (out of proportion to the injury), pallor, paresthesia, pulselessness, and paralysis. However, pulselessness is a late sign. The priority nursing intervention is to notify the physician immediately. Early recognition and prompt intervention are crucial to prevent irreversible muscle and nerve damage. Elevating the extremity can worsen compartment syndrome by decreasing arterial blood flow. Applying ice can also constrict blood vessels and further compromise circulation. Administering pain medication is important, but it does not address the underlying problem of increased compartment pressure. Therefore, the most appropriate initial action is to notify the physician immediately to facilitate prompt diagnosis and potential surgical intervention (fasciotomy) to relieve the pressure.
-
Question 12 of 30
12. Question
A 35-year-old patient is being discharged home after sustaining a fractured tibia and having a long leg cast applied. Which of the following instructions is MOST important for the nurse to emphasize to the patient regarding cast care?
Correct
The scenario involves a patient with a cast on their lower extremity who is being discharged home. The patient needs education on cast care to prevent complications such as skin breakdown, infection, and impaired circulation. The MOST important instruction to emphasize is to report any signs of impaired circulation, such as increased pain, swelling, numbness, tingling, or changes in skin color. These symptoms can indicate compartment syndrome or neurovascular compromise, which require immediate medical attention to prevent permanent damage. While keeping the cast dry, elevating the extremity, and performing range-of-motion exercises are all important aspects of cast care, they are secondary to recognizing and reporting signs of impaired circulation. Delaying treatment for compartment syndrome or neurovascular compromise can have serious consequences, including loss of limb function. The other instructions are important for overall cast care and comfort, but the priority is to ensure the patient understands the signs of a potentially limb-threatening complication.
Incorrect
The scenario involves a patient with a cast on their lower extremity who is being discharged home. The patient needs education on cast care to prevent complications such as skin breakdown, infection, and impaired circulation. The MOST important instruction to emphasize is to report any signs of impaired circulation, such as increased pain, swelling, numbness, tingling, or changes in skin color. These symptoms can indicate compartment syndrome or neurovascular compromise, which require immediate medical attention to prevent permanent damage. While keeping the cast dry, elevating the extremity, and performing range-of-motion exercises are all important aspects of cast care, they are secondary to recognizing and reporting signs of impaired circulation. Delaying treatment for compartment syndrome or neurovascular compromise can have serious consequences, including loss of limb function. The other instructions are important for overall cast care and comfort, but the priority is to ensure the patient understands the signs of a potentially limb-threatening complication.
-
Question 13 of 30
13. Question
An orthopedic nurse is caring for a patient who is two days post-operative from a total knee arthroplasty (TKA). Which of the following nursing interventions is MOST important for preventing venous thromboembolism (VTE) in this patient?
Correct
The question explores the nurse’s role in preventing venous thromboembolism (VTE) in a patient following a total knee arthroplasty (TKA). Mechanical prophylaxis, such as the use of sequential compression devices (SCDs), is a crucial component of VTE prevention in this population. SCDs apply intermittent pressure to the legs, promoting venous return and reducing the risk of blood clot formation. Proper application and consistent use of SCDs are essential for their effectiveness. The nurse should ensure that the SCDs are correctly sized and fitted to the patient’s legs, and that they are applied snugly but not too tightly. The patient should be instructed to wear the SCDs continuously, except during bathing or ambulation. The nurse should also regularly assess the patient’s skin for any signs of irritation or breakdown from the SCDs. While pharmacological prophylaxis (e.g., anticoagulants) is also commonly used, it is not the only measure to prevent VTE. Encouraging ambulation is important, but it is not a substitute for mechanical prophylaxis. Elevating the legs can help reduce swelling, but it does not directly prevent VTE.
Incorrect
The question explores the nurse’s role in preventing venous thromboembolism (VTE) in a patient following a total knee arthroplasty (TKA). Mechanical prophylaxis, such as the use of sequential compression devices (SCDs), is a crucial component of VTE prevention in this population. SCDs apply intermittent pressure to the legs, promoting venous return and reducing the risk of blood clot formation. Proper application and consistent use of SCDs are essential for their effectiveness. The nurse should ensure that the SCDs are correctly sized and fitted to the patient’s legs, and that they are applied snugly but not too tightly. The patient should be instructed to wear the SCDs continuously, except during bathing or ambulation. The nurse should also regularly assess the patient’s skin for any signs of irritation or breakdown from the SCDs. While pharmacological prophylaxis (e.g., anticoagulants) is also commonly used, it is not the only measure to prevent VTE. Encouraging ambulation is important, but it is not a substitute for mechanical prophylaxis. Elevating the legs can help reduce swelling, but it does not directly prevent VTE.
-
Question 14 of 30
14. Question
A 25-year-old male presents to the emergency department after falling off his skateboard. He reports pain and tenderness in his wrist, particularly in the anatomical snuffbox. Radiographs are initially negative for fracture, but the physician suspects a scaphoid fracture. Which of the following instructions is the MOST important to provide to the patient at discharge, considering the potential complications associated with scaphoid fractures and the need for appropriate follow-up? The instructions should emphasize the unique aspects of scaphoid fracture management.
Correct
The scenario describes a patient with a suspected scaphoid fracture following a fall. The key is to understand the unique characteristics of scaphoid fractures, including the potential for nonunion and avascular necrosis due to the bone’s precarious blood supply. While all options address important aspects of fracture management, the most crucial is to emphasize the importance of strict adherence to immobilization and follow-up appointments. Advising the patient that the pain will subside in a few days (option b) is inaccurate and could lead to delayed treatment. Telling the patient to ice and elevate the wrist (option c) is appropriate for initial management but doesn’t address the long-term implications. Recommending over-the-counter pain medication (option d) is helpful for pain relief but doesn’t emphasize the importance of follow-up. The correct approach is to emphasize the importance of strict adherence to immobilization with a cast or splint to promote healing and prevent nonunion, the need for regular follow-up appointments with an orthopedic surgeon to monitor fracture healing and assess for complications, and the potential for avascular necrosis if the fracture is not properly treated. This education ensures the patient understands the seriousness of the injury and the importance of following the treatment plan.
Incorrect
The scenario describes a patient with a suspected scaphoid fracture following a fall. The key is to understand the unique characteristics of scaphoid fractures, including the potential for nonunion and avascular necrosis due to the bone’s precarious blood supply. While all options address important aspects of fracture management, the most crucial is to emphasize the importance of strict adherence to immobilization and follow-up appointments. Advising the patient that the pain will subside in a few days (option b) is inaccurate and could lead to delayed treatment. Telling the patient to ice and elevate the wrist (option c) is appropriate for initial management but doesn’t address the long-term implications. Recommending over-the-counter pain medication (option d) is helpful for pain relief but doesn’t emphasize the importance of follow-up. The correct approach is to emphasize the importance of strict adherence to immobilization with a cast or splint to promote healing and prevent nonunion, the need for regular follow-up appointments with an orthopedic surgeon to monitor fracture healing and assess for complications, and the potential for avascular necrosis if the fracture is not properly treated. This education ensures the patient understands the seriousness of the injury and the importance of following the treatment plan.
-
Question 15 of 30
15. Question
A 50-year-old male presents to the emergency department complaining of severe lower back pain that started suddenly two days ago. He also reports numbness and tingling in his legs, bowel and bladder dysfunction, and saddle anesthesia. What is the MOST appropriate initial nursing action?
Correct
The correct answer emphasizes the need for a comprehensive assessment that includes a thorough history, physical examination, and appropriate diagnostic imaging to differentiate between various causes of back pain. It recognizes that cauda equina syndrome is a serious condition that requires prompt diagnosis and treatment. The other options represent incomplete or potentially misleading approaches. Recommending over-the-counter pain medication and rest without further evaluation could delay diagnosis and treatment of cauda equina syndrome. Referring the patient to a chiropractor without a clear diagnosis could exacerbate the condition and delay appropriate treatment. Assuming that the pain is due to muscle strain without further investigation could lead to misdiagnosis and inappropriate treatment. Back pain can be caused by a variety of factors, including muscle strain, herniated disc, spinal stenosis, and cauda equina syndrome. A thorough assessment is essential to determine the underlying cause of the pain and guide appropriate treatment.
Incorrect
The correct answer emphasizes the need for a comprehensive assessment that includes a thorough history, physical examination, and appropriate diagnostic imaging to differentiate between various causes of back pain. It recognizes that cauda equina syndrome is a serious condition that requires prompt diagnosis and treatment. The other options represent incomplete or potentially misleading approaches. Recommending over-the-counter pain medication and rest without further evaluation could delay diagnosis and treatment of cauda equina syndrome. Referring the patient to a chiropractor without a clear diagnosis could exacerbate the condition and delay appropriate treatment. Assuming that the pain is due to muscle strain without further investigation could lead to misdiagnosis and inappropriate treatment. Back pain can be caused by a variety of factors, including muscle strain, herniated disc, spinal stenosis, and cauda equina syndrome. A thorough assessment is essential to determine the underlying cause of the pain and guide appropriate treatment.
-
Question 16 of 30
16. Question
A 32-year-old male sustained a closed femoral shaft fracture in a motor vehicle accident. He underwent open reduction and internal fixation with an intramedullary nail 36 hours ago. The patient is now exhibiting increased dyspnea, with an oxygen saturation of 88% on 6L of oxygen via nasal cannula. Auscultation reveals bilateral diffuse crackles. A chest X-ray shows bilateral pulmonary infiltrates. The patient is afebrile and has no productive cough. He has no known history of cardiac disease. Which of the following conditions is the MOST likely cause of the patient’s current respiratory distress, and what immediate nursing actions are paramount?
Correct
The scenario describes a patient with a femoral shaft fracture being treated with an intramedullary nail. Postoperatively, the patient exhibits signs and symptoms suggestive of acute respiratory distress syndrome (ARDS), including dyspnea, hypoxemia unresponsive to oxygen supplementation, and bilateral pulmonary infiltrates on chest X-ray. The key concept here is recognizing the potential for fat embolism syndrome (FES) following long bone fractures, which can lead to ARDS. Fat embolism syndrome occurs when fat globules are released from the bone marrow into the circulation following a fracture. These fat globules can travel to the lungs and cause inflammation and capillary damage, leading to impaired gas exchange and ARDS. The time frame of 24-72 hours post-fracture fixation is consistent with the typical presentation of FES. While other conditions like pulmonary embolism (PE) and pneumonia are possible, the constellation of symptoms, the recent long bone fracture, and the timing strongly suggest FES. While a PE could present with similar symptoms, it’s less likely to be directly related to the fracture itself in this timeframe. Pneumonia would typically present with fever, productive cough, and localized infiltrates initially, rather than the diffuse bilateral infiltrates seen in ARDS secondary to FES. Myocardial infarction, while also serious, is less directly linked to the orthopedic procedure and the specific respiratory findings. Therefore, recognizing FES and initiating appropriate management, including respiratory support and monitoring for further complications, is crucial in this situation.
Incorrect
The scenario describes a patient with a femoral shaft fracture being treated with an intramedullary nail. Postoperatively, the patient exhibits signs and symptoms suggestive of acute respiratory distress syndrome (ARDS), including dyspnea, hypoxemia unresponsive to oxygen supplementation, and bilateral pulmonary infiltrates on chest X-ray. The key concept here is recognizing the potential for fat embolism syndrome (FES) following long bone fractures, which can lead to ARDS. Fat embolism syndrome occurs when fat globules are released from the bone marrow into the circulation following a fracture. These fat globules can travel to the lungs and cause inflammation and capillary damage, leading to impaired gas exchange and ARDS. The time frame of 24-72 hours post-fracture fixation is consistent with the typical presentation of FES. While other conditions like pulmonary embolism (PE) and pneumonia are possible, the constellation of symptoms, the recent long bone fracture, and the timing strongly suggest FES. While a PE could present with similar symptoms, it’s less likely to be directly related to the fracture itself in this timeframe. Pneumonia would typically present with fever, productive cough, and localized infiltrates initially, rather than the diffuse bilateral infiltrates seen in ARDS secondary to FES. Myocardial infarction, while also serious, is less directly linked to the orthopedic procedure and the specific respiratory findings. Therefore, recognizing FES and initiating appropriate management, including respiratory support and monitoring for further complications, is crucial in this situation.
-
Question 17 of 30
17. Question
A 45-year-old male with a well-documented history of opioid use disorder (OUD), currently managed with buprenorphine/naloxone, presents for elective total hip arthroplasty due to severe osteoarthritis. Pre-operative planning is underway. Which of the following represents the MOST appropriate and evidence-based approach to post-operative pain management for this patient?
Correct
The correct answer addresses the complexities of pain management in patients with a history of opioid use disorder (OUD). While opioids may be necessary for acute pain control following orthopedic surgery, they should be used cautiously and in conjunction with non-opioid analgesics and other pain management strategies. Regional anesthesia techniques, such as nerve blocks, can be highly effective in reducing post-operative pain and minimizing the need for opioids. Non-opioid analgesics, such as NSAIDs and acetaminophen, can also help to manage pain and reduce opioid consumption. A multimodal approach to pain management, which combines different types of analgesics and non-pharmacological interventions, is generally recommended for patients with OUD. A pain management specialist can provide valuable expertise in developing an individualized pain management plan. The scenario requires recognizing the challenges of pain management in patients with OUD and understanding the importance of a multimodal approach that minimizes opioid use while providing adequate pain relief. The nurse must be able to advocate for the patient’s needs and collaborate with other healthcare professionals to develop a safe and effective pain management plan.
Incorrect
The correct answer addresses the complexities of pain management in patients with a history of opioid use disorder (OUD). While opioids may be necessary for acute pain control following orthopedic surgery, they should be used cautiously and in conjunction with non-opioid analgesics and other pain management strategies. Regional anesthesia techniques, such as nerve blocks, can be highly effective in reducing post-operative pain and minimizing the need for opioids. Non-opioid analgesics, such as NSAIDs and acetaminophen, can also help to manage pain and reduce opioid consumption. A multimodal approach to pain management, which combines different types of analgesics and non-pharmacological interventions, is generally recommended for patients with OUD. A pain management specialist can provide valuable expertise in developing an individualized pain management plan. The scenario requires recognizing the challenges of pain management in patients with OUD and understanding the importance of a multimodal approach that minimizes opioid use while providing adequate pain relief. The nurse must be able to advocate for the patient’s needs and collaborate with other healthcare professionals to develop a safe and effective pain management plan.
-
Question 18 of 30
18. Question
An 82-year-old patient with advanced dementia is admitted for a hip fracture following a fall. The patient is non-verbal and unable to participate in decision-making. A family member arrives, presenting a power of attorney document, and insists that the patient undergo immediate surgical repair of the hip fracture. As the orthopedic nurse, you review the patient’s chart and note that the patient has multiple comorbidities, including severe heart disease and chronic kidney disease, which significantly increase the risks associated with surgery. You also have concerns about the validity of the power of attorney document presented by the family member. Considering the ethical and legal responsibilities in this complex scenario, what is the most appropriate initial action for the orthopedic nurse to take?
Correct
The correct approach to this scenario involves understanding the ethical and legal considerations surrounding informed consent, particularly in the context of a patient with cognitive impairment. The nurse’s primary responsibility is to advocate for the patient and ensure their rights are protected. This includes verifying the validity of the power of attorney, which means confirming it is legally sound and grants the representative the authority to make healthcare decisions. If the power of attorney is valid, the nurse must then ensure the representative fully understands the proposed surgical procedure, its risks, benefits, and alternatives. The representative’s decision should be based on what they believe the patient would have wanted, aligning with the principle of substituted judgment. Simply proceeding with the surgery based on the representative’s consent without ensuring their understanding and the validity of their authority would be a violation of the patient’s autonomy and could lead to legal repercussions. Consulting hospital ethics committee is crucial to address ethical dilemmas, clarify legal uncertainties, and ensure the decision-making process aligns with ethical principles and legal standards. The ethics committee can provide guidance on how to best protect the patient’s interests and ensure all parties involved are acting in accordance with ethical and legal obligations. This multidisciplinary approach ensures a comprehensive review of the situation, promoting a decision that respects the patient’s rights and values. It is important to document all steps taken, including verification of the power of attorney, discussions with the representative, and consultation with the ethics committee, to provide a clear record of the decision-making process and demonstrate adherence to ethical and legal standards.
Incorrect
The correct approach to this scenario involves understanding the ethical and legal considerations surrounding informed consent, particularly in the context of a patient with cognitive impairment. The nurse’s primary responsibility is to advocate for the patient and ensure their rights are protected. This includes verifying the validity of the power of attorney, which means confirming it is legally sound and grants the representative the authority to make healthcare decisions. If the power of attorney is valid, the nurse must then ensure the representative fully understands the proposed surgical procedure, its risks, benefits, and alternatives. The representative’s decision should be based on what they believe the patient would have wanted, aligning with the principle of substituted judgment. Simply proceeding with the surgery based on the representative’s consent without ensuring their understanding and the validity of their authority would be a violation of the patient’s autonomy and could lead to legal repercussions. Consulting hospital ethics committee is crucial to address ethical dilemmas, clarify legal uncertainties, and ensure the decision-making process aligns with ethical principles and legal standards. The ethics committee can provide guidance on how to best protect the patient’s interests and ensure all parties involved are acting in accordance with ethical and legal obligations. This multidisciplinary approach ensures a comprehensive review of the situation, promoting a decision that respects the patient’s rights and values. It is important to document all steps taken, including verification of the power of attorney, discussions with the representative, and consultation with the ethics committee, to provide a clear record of the decision-making process and demonstrate adherence to ethical and legal standards.
-
Question 19 of 30
19. Question
A 70-year-old male is two days post-operative following a total knee arthroplasty (TKA). He reports sudden onset of shortness of breath and sharp, stabbing chest pain that worsens with deep inspiration. He denies fever, chills, or productive cough. Which of the following complications should the orthopedic nurse suspect FIRST?
Correct
The question tests knowledge of surgical procedures in orthopedics, specifically total knee arthroplasty (TKA), and potential postoperative complications, with a focus on deep vein thrombosis (DVT) and pulmonary embolism (PE). TKA carries a significant risk of thromboembolic events due to surgical trauma, prolonged immobilization, and hypercoagulability. Signs and symptoms of DVT include unilateral leg swelling, pain, warmth, and redness. A PE occurs when a blood clot travels to the lungs, obstructing pulmonary blood flow. Symptoms of PE include sudden onset of shortness of breath, chest pain (often pleuritic), cough, and sometimes hemoptysis (coughing up blood). The Virchow’s triad, which includes venous stasis, hypercoagulability, and endothelial injury, are the main factors that contribute to thrombus formation. The patient’s report of sudden onset shortness of breath and pleuritic chest pain is highly suggestive of a PE. While wound drainage and fever could indicate infection, and calf pain could indicate DVT, the combination of respiratory symptoms points most strongly to a PE. Hypertension is not directly related to PE.
Incorrect
The question tests knowledge of surgical procedures in orthopedics, specifically total knee arthroplasty (TKA), and potential postoperative complications, with a focus on deep vein thrombosis (DVT) and pulmonary embolism (PE). TKA carries a significant risk of thromboembolic events due to surgical trauma, prolonged immobilization, and hypercoagulability. Signs and symptoms of DVT include unilateral leg swelling, pain, warmth, and redness. A PE occurs when a blood clot travels to the lungs, obstructing pulmonary blood flow. Symptoms of PE include sudden onset of shortness of breath, chest pain (often pleuritic), cough, and sometimes hemoptysis (coughing up blood). The Virchow’s triad, which includes venous stasis, hypercoagulability, and endothelial injury, are the main factors that contribute to thrombus formation. The patient’s report of sudden onset shortness of breath and pleuritic chest pain is highly suggestive of a PE. While wound drainage and fever could indicate infection, and calf pain could indicate DVT, the combination of respiratory symptoms points most strongly to a PE. Hypertension is not directly related to PE.
-
Question 20 of 30
20. Question
A 52-year-old male presents to the orthopedic clinic with persistent drainage from a surgical wound site six weeks after undergoing open reduction and internal fixation (ORIF) of a tibial shaft fracture. He was initially treated with intravenous (IV) antibiotics for presumed surgical site infection, but the drainage has not resolved. Radiographs show complete union of the tibial fracture with well-positioned plates and screws. Bone biopsy confirms the presence of osteomyelitis. Which of the following is the MOST appropriate next step in managing this patient’s osteomyelitis?
Correct
This question focuses on the complexities of managing osteomyelitis, a bone infection, particularly in the context of hardware retention. The presence of hardware (plates and screws) significantly complicates the treatment of osteomyelitis because the hardware can act as a biofilm, providing a surface for bacteria to adhere to and making it difficult for antibiotics to eradicate the infection. The initial step in managing hardware-associated osteomyelitis is typically surgical debridement. This involves removing all infected tissue, including any necrotic bone or soft tissue. However, the decision to remove or retain the hardware is complex and depends on several factors, including the stability of the fracture, the duration of the infection, and the patient’s overall condition. In this scenario, the fracture is well-healed, indicating that the hardware is no longer essential for fracture stability. Given the chronic nature of the infection (present for 6 weeks despite IV antibiotics), the most appropriate approach is to remove the hardware, perform thorough debridement of all infected tissue, and obtain bone cultures. The bone cultures will guide the selection of appropriate long-term antibiotic therapy. While continuing IV antibiotics alone might temporarily suppress the infection, it is unlikely to eradicate it completely in the presence of hardware. Irrigation and debridement without hardware removal may provide some benefit, but the risk of persistent infection remains high. Immediate re-implantation of new hardware is contraindicated in the setting of active infection.
Incorrect
This question focuses on the complexities of managing osteomyelitis, a bone infection, particularly in the context of hardware retention. The presence of hardware (plates and screws) significantly complicates the treatment of osteomyelitis because the hardware can act as a biofilm, providing a surface for bacteria to adhere to and making it difficult for antibiotics to eradicate the infection. The initial step in managing hardware-associated osteomyelitis is typically surgical debridement. This involves removing all infected tissue, including any necrotic bone or soft tissue. However, the decision to remove or retain the hardware is complex and depends on several factors, including the stability of the fracture, the duration of the infection, and the patient’s overall condition. In this scenario, the fracture is well-healed, indicating that the hardware is no longer essential for fracture stability. Given the chronic nature of the infection (present for 6 weeks despite IV antibiotics), the most appropriate approach is to remove the hardware, perform thorough debridement of all infected tissue, and obtain bone cultures. The bone cultures will guide the selection of appropriate long-term antibiotic therapy. While continuing IV antibiotics alone might temporarily suppress the infection, it is unlikely to eradicate it completely in the presence of hardware. Irrigation and debridement without hardware removal may provide some benefit, but the risk of persistent infection remains high. Immediate re-implantation of new hardware is contraindicated in the setting of active infection.
-
Question 21 of 30
21. Question
An orthopedic nursing team is tasked with improving patient outcomes following total hip arthroplasty (THA). Which of the following strategies would be most effective in implementing a comprehensive quality improvement (QI) initiative to enhance patient care and minimize complications, considering the multifaceted nature of orthopedic outcomes and the need for systematic approaches to improve healthcare processes?
Correct
The question addresses the principles of quality improvement in orthopedic nursing. Quality improvement (QI) aims to systematically improve healthcare processes and outcomes. A key component of QI is data collection and analysis to identify areas for improvement and measure the impact of interventions. This involves collecting data on relevant indicators, such as surgical site infection rates, patient satisfaction scores, and adherence to evidence-based guidelines. Analyzing this data can help identify trends, patterns, and areas where performance is not meeting expectations. Implementing evidence-based practices is another essential aspect of QI. This involves using the best available research evidence to guide clinical decision-making and improve patient care. Examples of evidence-based practices in orthopedic nursing include using chlorhexidine gluconate (CHG) for preoperative skin preparation to reduce surgical site infections and implementing early mobilization protocols to prevent complications after surgery. Patient safety initiatives are also crucial for QI in orthopedic nursing. This involves implementing strategies to prevent errors, reduce harm, and create a culture of safety. Examples of patient safety initiatives include using checklists to ensure proper surgical site verification and implementing fall prevention programs to reduce the risk of falls in orthopedic patients.
Incorrect
The question addresses the principles of quality improvement in orthopedic nursing. Quality improvement (QI) aims to systematically improve healthcare processes and outcomes. A key component of QI is data collection and analysis to identify areas for improvement and measure the impact of interventions. This involves collecting data on relevant indicators, such as surgical site infection rates, patient satisfaction scores, and adherence to evidence-based guidelines. Analyzing this data can help identify trends, patterns, and areas where performance is not meeting expectations. Implementing evidence-based practices is another essential aspect of QI. This involves using the best available research evidence to guide clinical decision-making and improve patient care. Examples of evidence-based practices in orthopedic nursing include using chlorhexidine gluconate (CHG) for preoperative skin preparation to reduce surgical site infections and implementing early mobilization protocols to prevent complications after surgery. Patient safety initiatives are also crucial for QI in orthopedic nursing. This involves implementing strategies to prevent errors, reduce harm, and create a culture of safety. Examples of patient safety initiatives include using checklists to ensure proper surgical site verification and implementing fall prevention programs to reduce the risk of falls in orthopedic patients.
-
Question 22 of 30
22. Question
A 62-year-old patient with a closed tibial fracture develops acute compartment syndrome in their lower leg. The orthopedic surgeon recommends an immediate fasciotomy. The patient, alert and oriented, expresses a firm refusal of the surgery, stating they “don’t want any more operations.” The patient has been thoroughly educated about the risks of compartment syndrome, including potential limb loss, by the surgical team. As a certified orthopedic nurse, what is the MOST ethically and legally sound initial action you should take? This situation requires navigating patient autonomy, potential harm, and legal considerations within the scope of orthopedic nursing practice. Consider the immediate steps to protect the patient’s rights while ensuring their well-being and minimizing potential legal repercussions for the healthcare team. Prioritize actions that facilitate informed decision-making and ethical care delivery in this complex scenario.
Correct
The correct approach involves understanding the pathophysiology of compartment syndrome and the principles of ethical decision-making in healthcare. Compartment syndrome is a condition where increased pressure within a confined space (muscle compartment) compromises the circulation and function of the tissues within that space. This can lead to irreversible muscle and nerve damage if not promptly treated. A fasciotomy, a surgical procedure to release the pressure, is the definitive treatment. In this scenario, the patient is refusing the recommended treatment (fasciotomy). The nurse’s primary ethical obligations are to respect patient autonomy (the right to make informed decisions about their own care) and to act in the patient’s best interest (beneficence). However, these principles can sometimes conflict. While respecting the patient’s refusal, the nurse also has a responsibility to ensure the patient is fully informed about the potential consequences of their decision, including permanent disability. Simply respecting the refusal without further action could be considered negligence, as it could lead to preventable harm. Attempting to obtain a court order to force the procedure is generally not the first step, especially in a non-emergent situation where the patient retains decision-making capacity. The initial and most appropriate action is to engage in a thorough discussion with the patient, providing clear and understandable information about the risks and benefits of both the fasciotomy and the refusal of the procedure. This discussion should be documented meticulously. Consulting with the ethics committee is also appropriate, but not before engaging in a detailed discussion with the patient.
Incorrect
The correct approach involves understanding the pathophysiology of compartment syndrome and the principles of ethical decision-making in healthcare. Compartment syndrome is a condition where increased pressure within a confined space (muscle compartment) compromises the circulation and function of the tissues within that space. This can lead to irreversible muscle and nerve damage if not promptly treated. A fasciotomy, a surgical procedure to release the pressure, is the definitive treatment. In this scenario, the patient is refusing the recommended treatment (fasciotomy). The nurse’s primary ethical obligations are to respect patient autonomy (the right to make informed decisions about their own care) and to act in the patient’s best interest (beneficence). However, these principles can sometimes conflict. While respecting the patient’s refusal, the nurse also has a responsibility to ensure the patient is fully informed about the potential consequences of their decision, including permanent disability. Simply respecting the refusal without further action could be considered negligence, as it could lead to preventable harm. Attempting to obtain a court order to force the procedure is generally not the first step, especially in a non-emergent situation where the patient retains decision-making capacity. The initial and most appropriate action is to engage in a thorough discussion with the patient, providing clear and understandable information about the risks and benefits of both the fasciotomy and the refusal of the procedure. This discussion should be documented meticulously. Consulting with the ethics committee is also appropriate, but not before engaging in a detailed discussion with the patient.
-
Question 23 of 30
23. Question
An orthopedic nurse is caring for a 68-year-old patient recovering from a total knee arthroplasty. The patient reports persistent, severe pain (8/10 on the pain scale) despite receiving the prescribed opioid analgesic every 4 hours. The patient has a documented history of opioid use disorder from 10 years prior, which was reportedly managed with outpatient therapy. The patient is demanding increased doses of the opioid medication and expresses anger and frustration when the nurse attempts to explore alternative pain management strategies. The physician is hesitant to increase the opioid dosage due to the patient’s history and potential risk of relapse. Which of the following actions is the MOST appropriate for the orthopedic nurse to take in this situation, balancing ethical considerations, legal responsibilities, and patient advocacy?
Correct
The scenario presents a complex ethical dilemma involving a patient with chronic pain, potential opioid use disorder, and the nurse’s role in advocating for appropriate pain management while considering patient safety and legal regulations. The most appropriate course of action involves several steps. First, the nurse must acknowledge the patient’s reported pain level and conduct a comprehensive pain assessment. This includes evaluating the pain’s characteristics, intensity, aggravating/alleviating factors, and the impact on the patient’s functional status. Next, the nurse needs to review the patient’s medical history, paying close attention to any documented history of substance use disorder, previous pain management strategies, and current medications. Considering the potential for opioid use disorder, the nurse should collaborate with the healthcare team, including the physician and potentially a pain management specialist or addiction specialist, to develop a safe and effective pain management plan. This plan should prioritize non-opioid pain management strategies whenever possible, such as physical therapy, occupational therapy, NSAIDs, acetaminophen, and adjuvant medications like gabapentin or pregabalin. If opioids are deemed necessary, they should be prescribed at the lowest effective dose for the shortest duration possible, with careful monitoring for signs of misuse or addiction. The nurse must also provide the patient with comprehensive education regarding the risks and benefits of opioid therapy, including the potential for dependence, addiction, and adverse effects. Furthermore, the nurse should advocate for the patient’s access to resources for addiction treatment and support if needed, such as counseling, support groups, and medication-assisted treatment. Documenting all assessments, interventions, and communication with the healthcare team and the patient is crucial to ensure legal and ethical compliance. This approach balances the patient’s right to pain relief with the nurse’s responsibility to prevent harm and promote patient safety.
Incorrect
The scenario presents a complex ethical dilemma involving a patient with chronic pain, potential opioid use disorder, and the nurse’s role in advocating for appropriate pain management while considering patient safety and legal regulations. The most appropriate course of action involves several steps. First, the nurse must acknowledge the patient’s reported pain level and conduct a comprehensive pain assessment. This includes evaluating the pain’s characteristics, intensity, aggravating/alleviating factors, and the impact on the patient’s functional status. Next, the nurse needs to review the patient’s medical history, paying close attention to any documented history of substance use disorder, previous pain management strategies, and current medications. Considering the potential for opioid use disorder, the nurse should collaborate with the healthcare team, including the physician and potentially a pain management specialist or addiction specialist, to develop a safe and effective pain management plan. This plan should prioritize non-opioid pain management strategies whenever possible, such as physical therapy, occupational therapy, NSAIDs, acetaminophen, and adjuvant medications like gabapentin or pregabalin. If opioids are deemed necessary, they should be prescribed at the lowest effective dose for the shortest duration possible, with careful monitoring for signs of misuse or addiction. The nurse must also provide the patient with comprehensive education regarding the risks and benefits of opioid therapy, including the potential for dependence, addiction, and adverse effects. Furthermore, the nurse should advocate for the patient’s access to resources for addiction treatment and support if needed, such as counseling, support groups, and medication-assisted treatment. Documenting all assessments, interventions, and communication with the healthcare team and the patient is crucial to ensure legal and ethical compliance. This approach balances the patient’s right to pain relief with the nurse’s responsibility to prevent harm and promote patient safety.
-
Question 24 of 30
24. Question
A 72-year-old female patient with a history of osteoporosis has been taking alendronate (a bisphosphonate) for the past 8 years. She presents to the orthopedic clinic complaining of new-onset, persistent pain in her right thigh that has been present for the last several weeks. She denies any recent trauma or falls. Upon examination, the orthopedic surgeon suspects a possible atypical femur fracture (AFF). Which of the following pathophysiological mechanisms is MOST directly associated with the increased risk of AFFs in patients on long-term bisphosphonate therapy, and what crucial assessment finding should the orthopedic nurse prioritize based on this understanding?
Correct
The correct answer involves understanding the interplay between bisphosphonates, bone remodeling, and atypical femur fractures (AFFs). Bisphosphonates, while effective in increasing bone density and reducing the risk of osteoporotic fractures, can suppress bone turnover excessively. This suppression inhibits the removal of microdamage that accumulates in the bone over time. The prolonged suppression of bone remodeling, particularly in the femoral shaft, can lead to increased bone brittleness and susceptibility to stress fractures. These stress fractures often start in the lateral cortex of the femur and can propagate, eventually leading to an atypical femur fracture. These fractures are termed “atypical” because they differ from typical osteoporotic fractures in their location, fracture pattern (often transverse or short oblique), and association with bisphosphonate use. The duration of bisphosphonate use is a significant risk factor, with longer durations associated with a higher risk of AFFs. Therefore, recognizing this potential adverse effect and monitoring patients on long-term bisphosphonate therapy for prodromal symptoms (e.g., thigh or groin pain) is crucial for orthopedic nurses. Discontinuing bisphosphonates after 5 years in patients at low risk for fracture is a strategy to mitigate the risk of AFFs. The nurse must also educate patients about the signs and symptoms of AFFs and the importance of reporting any new thigh or groin pain to their healthcare provider. Other options are less directly related to the mechanism by which bisphosphonates contribute to AFFs. While bisphosphonates can affect calcium homeostasis and bone mineral density, the primary mechanism leading to AFFs is the suppression of bone remodeling. Similarly, while bisphosphonates can affect osteoblast activity, the critical factor in AFFs is the overall suppression of bone turnover, including osteoclast activity. The interaction with vitamin D metabolism is less directly related to the pathogenesis of AFFs compared to the suppression of bone remodeling.
Incorrect
The correct answer involves understanding the interplay between bisphosphonates, bone remodeling, and atypical femur fractures (AFFs). Bisphosphonates, while effective in increasing bone density and reducing the risk of osteoporotic fractures, can suppress bone turnover excessively. This suppression inhibits the removal of microdamage that accumulates in the bone over time. The prolonged suppression of bone remodeling, particularly in the femoral shaft, can lead to increased bone brittleness and susceptibility to stress fractures. These stress fractures often start in the lateral cortex of the femur and can propagate, eventually leading to an atypical femur fracture. These fractures are termed “atypical” because they differ from typical osteoporotic fractures in their location, fracture pattern (often transverse or short oblique), and association with bisphosphonate use. The duration of bisphosphonate use is a significant risk factor, with longer durations associated with a higher risk of AFFs. Therefore, recognizing this potential adverse effect and monitoring patients on long-term bisphosphonate therapy for prodromal symptoms (e.g., thigh or groin pain) is crucial for orthopedic nurses. Discontinuing bisphosphonates after 5 years in patients at low risk for fracture is a strategy to mitigate the risk of AFFs. The nurse must also educate patients about the signs and symptoms of AFFs and the importance of reporting any new thigh or groin pain to their healthcare provider. Other options are less directly related to the mechanism by which bisphosphonates contribute to AFFs. While bisphosphonates can affect calcium homeostasis and bone mineral density, the primary mechanism leading to AFFs is the suppression of bone remodeling. Similarly, while bisphosphonates can affect osteoblast activity, the critical factor in AFFs is the overall suppression of bone turnover, including osteoclast activity. The interaction with vitamin D metabolism is less directly related to the pathogenesis of AFFs compared to the suppression of bone remodeling.
-
Question 25 of 30
25. Question
A 68-year-old male with a history of poorly controlled type 2 diabetes (HbA1c 8.5%), a BMI of 34 kg/m², and a 40-pack-year smoking history is scheduled for a total knee arthroplasty (TKA) in two weeks. During the preoperative assessment, the orthopedic nurse identifies the patient as being at high risk for developing a surgical site infection (SSI). Considering the patient’s comorbidities and the need to minimize the risk of SSI, which of the following interventions would be the MOST comprehensive and evidence-based approach to implement preoperatively? Assume all interventions are implemented in addition to standard preoperative procedures such as surgical site marking and verification of informed consent.
Correct
The scenario describes a patient at high risk for developing a surgical site infection (SSI) following a total knee arthroplasty (TKA). Several factors contribute to this risk: the patient’s history of poorly controlled diabetes, obesity (BMI > 30), and current smoking habit. These conditions impair wound healing and compromise the immune system, increasing susceptibility to infection. The primary goal is to prevent SSI through meticulous preoperative preparation and adherence to evidence-based practices. While all listed interventions are important in orthopedic nursing, the most impactful in this scenario focuses on optimizing the patient’s modifiable risk factors and implementing stringent infection control measures. Tight glycemic control is paramount. Elevated blood glucose levels provide a favorable environment for bacterial growth and impair leukocyte function, hindering the body’s ability to fight infection. Preoperative HbA1c testing and subsequent management with insulin or oral hypoglycemic agents is crucial. Chlorhexidine gluconate (CHG) bathing reduces the skin’s bacterial load. Preoperative showering with CHG washcloths for several days before surgery significantly decreases the risk of SSI by eliminating potential pathogens from the surgical site. Smoking cessation, even in the short term, can improve wound healing. Smoking impairs microcirculation and reduces oxygen delivery to tissues, delaying healing and increasing infection risk. Encouraging smoking cessation and providing resources can positively impact outcomes. While prophylactic antibiotics are essential, their effectiveness is maximized when combined with optimized patient factors and meticulous skin preparation. Early mobilization and pain management are crucial for postoperative recovery but are secondary to addressing the patient’s immediate infection risks preoperatively. Therefore, the most comprehensive approach involves a combination of optimizing blood glucose, CHG bathing, and promoting smoking cessation.
Incorrect
The scenario describes a patient at high risk for developing a surgical site infection (SSI) following a total knee arthroplasty (TKA). Several factors contribute to this risk: the patient’s history of poorly controlled diabetes, obesity (BMI > 30), and current smoking habit. These conditions impair wound healing and compromise the immune system, increasing susceptibility to infection. The primary goal is to prevent SSI through meticulous preoperative preparation and adherence to evidence-based practices. While all listed interventions are important in orthopedic nursing, the most impactful in this scenario focuses on optimizing the patient’s modifiable risk factors and implementing stringent infection control measures. Tight glycemic control is paramount. Elevated blood glucose levels provide a favorable environment for bacterial growth and impair leukocyte function, hindering the body’s ability to fight infection. Preoperative HbA1c testing and subsequent management with insulin or oral hypoglycemic agents is crucial. Chlorhexidine gluconate (CHG) bathing reduces the skin’s bacterial load. Preoperative showering with CHG washcloths for several days before surgery significantly decreases the risk of SSI by eliminating potential pathogens from the surgical site. Smoking cessation, even in the short term, can improve wound healing. Smoking impairs microcirculation and reduces oxygen delivery to tissues, delaying healing and increasing infection risk. Encouraging smoking cessation and providing resources can positively impact outcomes. While prophylactic antibiotics are essential, their effectiveness is maximized when combined with optimized patient factors and meticulous skin preparation. Early mobilization and pain management are crucial for postoperative recovery but are secondary to addressing the patient’s immediate infection risks preoperatively. Therefore, the most comprehensive approach involves a combination of optimizing blood glucose, CHG bathing, and promoting smoking cessation.
-
Question 26 of 30
26. Question
A 78-year-old female patient with a history of a surgically repaired intertrochanteric hip fracture presents to the clinic complaining of a sudden increase in pain in her hip over the past week. She reports difficulty ambulating and is now using a walker for support. She denies any recent falls or trauma. On examination, you note tenderness to palpation over the lateral hip and limited range of motion. She is afebrile, and there are no signs of infection. Which of the following is the MOST likely cause of her current symptoms?
Correct
The scenario describes a patient with a history of a hip fracture who is now experiencing increased pain and limited mobility. Given the patient’s history and symptoms, the most likely cause is hardware failure. Hardware failure can occur due to loosening, breakage, or migration of the implants used to fix the fracture. This can lead to instability and pain. Avascular necrosis (AVN) is a possible complication of hip fractures, but typically presents with a more gradual onset of pain and stiffness. Infection would typically present with signs of inflammation, such as fever, redness, and swelling. Heterotopic ossification (HO) can cause stiffness and pain, but is less likely to cause a sudden increase in pain and decreased mobility.
Incorrect
The scenario describes a patient with a history of a hip fracture who is now experiencing increased pain and limited mobility. Given the patient’s history and symptoms, the most likely cause is hardware failure. Hardware failure can occur due to loosening, breakage, or migration of the implants used to fix the fracture. This can lead to instability and pain. Avascular necrosis (AVN) is a possible complication of hip fractures, but typically presents with a more gradual onset of pain and stiffness. Infection would typically present with signs of inflammation, such as fever, redness, and swelling. Heterotopic ossification (HO) can cause stiffness and pain, but is less likely to cause a sudden increase in pain and decreased mobility.
-
Question 27 of 30
27. Question
A 72-year-old female with a history of osteoporosis has been taking oral alendronate (a bisphosphonate) for the past 8 years. She presents to the clinic complaining of persistent, dull aching pain in her right groin and anterior thigh for the past 3 months. She denies any recent trauma or falls. Physical examination reveals mild tenderness to palpation over the proximal femur. Radiographs reveal subtle cortical thickening on the lateral aspect of the femoral shaft, but no visible fracture line. Based on the patient’s history, clinical presentation, and radiographic findings, which of the following actions is the MOST appropriate next step in managing this patient’s care, considering the potential risk of atypical femur fracture (AFF) and current guidelines regarding bisphosphonate use? The patient has been diligent about taking her medication and also takes a daily multivitamin.
Correct
The question explores the complex interplay between bisphosphonates, atypical femur fractures (AFFs), and bone remodeling. Atypical femur fractures are stress fractures that occur in the subtrochanteric region or femoral shaft in patients taking bisphosphonates, medications used to treat osteoporosis. The prolonged suppression of bone turnover by bisphosphonates is thought to contribute to the accumulation of microdamage within the bone matrix, increasing the risk of AFFs. Bone remodeling is a continuous process involving bone resorption by osteoclasts and bone formation by osteoblasts. Bisphosphonates inhibit osteoclast activity, reducing bone resorption. While this increases bone mineral density (BMD), it also impairs the bone’s ability to repair microdamage. The accumulation of microdamage weakens the bone, predisposing it to AFFs. The duration of bisphosphonate use is a significant risk factor for AFFs. The longer a patient takes bisphosphonates, the greater the suppression of bone turnover and the accumulation of microdamage. The risk of AFFs increases significantly after 3-5 years of bisphosphonate use. Other factors, such as genetics, vitamin D deficiency, and certain medications, may also contribute to the risk of AFFs. When a patient on long-term bisphosphonate therapy presents with groin or thigh pain, it is crucial to consider the possibility of an impending or incomplete AFF. Radiographic evaluation, including X-rays of the entire femur, is essential to assess for cortical thickening, periosteal reaction, or a transverse fracture line. If an AFF is suspected, the bisphosphonate should be discontinued, and the patient should be referred to an orthopedic surgeon for further evaluation and management. Management strategies may include prophylactic fixation of the contralateral femur if radiographic evidence of an impending fracture is present. The goal is to prevent a complete fracture and minimize morbidity.
Incorrect
The question explores the complex interplay between bisphosphonates, atypical femur fractures (AFFs), and bone remodeling. Atypical femur fractures are stress fractures that occur in the subtrochanteric region or femoral shaft in patients taking bisphosphonates, medications used to treat osteoporosis. The prolonged suppression of bone turnover by bisphosphonates is thought to contribute to the accumulation of microdamage within the bone matrix, increasing the risk of AFFs. Bone remodeling is a continuous process involving bone resorption by osteoclasts and bone formation by osteoblasts. Bisphosphonates inhibit osteoclast activity, reducing bone resorption. While this increases bone mineral density (BMD), it also impairs the bone’s ability to repair microdamage. The accumulation of microdamage weakens the bone, predisposing it to AFFs. The duration of bisphosphonate use is a significant risk factor for AFFs. The longer a patient takes bisphosphonates, the greater the suppression of bone turnover and the accumulation of microdamage. The risk of AFFs increases significantly after 3-5 years of bisphosphonate use. Other factors, such as genetics, vitamin D deficiency, and certain medications, may also contribute to the risk of AFFs. When a patient on long-term bisphosphonate therapy presents with groin or thigh pain, it is crucial to consider the possibility of an impending or incomplete AFF. Radiographic evaluation, including X-rays of the entire femur, is essential to assess for cortical thickening, periosteal reaction, or a transverse fracture line. If an AFF is suspected, the bisphosphonate should be discontinued, and the patient should be referred to an orthopedic surgeon for further evaluation and management. Management strategies may include prophylactic fixation of the contralateral femur if radiographic evidence of an impending fracture is present. The goal is to prevent a complete fracture and minimize morbidity.
-
Question 28 of 30
28. Question
A 55-year-old female patient with a long-standing history of rheumatoid arthritis (RA) presents to the orthopedic clinic complaining of persistent neck pain and stiffness. On neurological examination, you note the presence of hyperreflexia in her lower extremities, clonus at the ankles, and a positive Babinski sign bilaterally. These findings are MOST indicative of which of the following complications of rheumatoid arthritis?
Correct
This scenario describes a patient with rheumatoid arthritis (RA) experiencing cervical spine instability, specifically atlantoaxial instability (AAI). RA is a chronic autoimmune disease that can affect the synovial joints, including those in the cervical spine. Inflammation and erosion of the ligaments and bone in the cervical spine can lead to instability, particularly at the atlantoaxial joint (C1-C2). Atlantoaxial instability can cause spinal cord compression, leading to neurological symptoms such as neck pain, stiffness, headaches, and, in severe cases, myelopathy (spinal cord dysfunction). Signs of myelopathy can include hyperreflexia (exaggerated reflexes), clonus (rhythmic muscle contractions), and a positive Babinski sign (extension of the big toe and fanning of the other toes in response to plantar stimulation). These findings indicate upper motor neuron involvement due to spinal cord compression. Loss of bowel and bladder control is a late sign of severe spinal cord compression. Decreased range of motion is a common symptom of RA but is not specific to cervical myelopathy. Peripheral neuropathy can cause numbness and tingling in the extremities, but it does not explain the upper motor neuron signs observed in this patient. Therefore, the presence of hyperreflexia, clonus, and a positive Babinski sign in a patient with RA and neck pain is most indicative of cervical myelopathy secondary to atlantoaxial instability.
Incorrect
This scenario describes a patient with rheumatoid arthritis (RA) experiencing cervical spine instability, specifically atlantoaxial instability (AAI). RA is a chronic autoimmune disease that can affect the synovial joints, including those in the cervical spine. Inflammation and erosion of the ligaments and bone in the cervical spine can lead to instability, particularly at the atlantoaxial joint (C1-C2). Atlantoaxial instability can cause spinal cord compression, leading to neurological symptoms such as neck pain, stiffness, headaches, and, in severe cases, myelopathy (spinal cord dysfunction). Signs of myelopathy can include hyperreflexia (exaggerated reflexes), clonus (rhythmic muscle contractions), and a positive Babinski sign (extension of the big toe and fanning of the other toes in response to plantar stimulation). These findings indicate upper motor neuron involvement due to spinal cord compression. Loss of bowel and bladder control is a late sign of severe spinal cord compression. Decreased range of motion is a common symptom of RA but is not specific to cervical myelopathy. Peripheral neuropathy can cause numbness and tingling in the extremities, but it does not explain the upper motor neuron signs observed in this patient. Therefore, the presence of hyperreflexia, clonus, and a positive Babinski sign in a patient with RA and neck pain is most indicative of cervical myelopathy secondary to atlantoaxial instability.
-
Question 29 of 30
29. Question
A 35-year-old male patient with a tibial shaft fracture is being discharged home with an external fixator. Which of the following instructions is MOST important for the nurse to provide regarding pin site care? The patient is otherwise healthy and has no known allergies. He is able to ambulate with crutches and is tolerating oral pain medication.
Correct
The scenario describes a patient with a tibial shaft fracture who is being discharged home with an external fixator. Patient education is crucial for ensuring proper care of the external fixator and preventing complications such as pin site infections. The patient needs to understand the importance of regular pin site care, proper cleaning techniques, and signs and symptoms of infection. While soaking the pin sites in hydrogen peroxide was a common practice in the past, it is no longer recommended as it can be cytotoxic to healthy tissue and may delay wound healing. Applying antibiotic ointment to the pin sites is also not routinely recommended, as it can contribute to antibiotic resistance. Covering the pin sites with occlusive dressings can create a moist environment that promotes bacterial growth and increases the risk of infection. The most appropriate instruction is to clean the pin sites daily with sterile saline solution and assess for signs of infection, such as redness, swelling, drainage, or increased pain. The patient should also be instructed to report any signs of infection to their healthcare provider immediately.
Incorrect
The scenario describes a patient with a tibial shaft fracture who is being discharged home with an external fixator. Patient education is crucial for ensuring proper care of the external fixator and preventing complications such as pin site infections. The patient needs to understand the importance of regular pin site care, proper cleaning techniques, and signs and symptoms of infection. While soaking the pin sites in hydrogen peroxide was a common practice in the past, it is no longer recommended as it can be cytotoxic to healthy tissue and may delay wound healing. Applying antibiotic ointment to the pin sites is also not routinely recommended, as it can contribute to antibiotic resistance. Covering the pin sites with occlusive dressings can create a moist environment that promotes bacterial growth and increases the risk of infection. The most appropriate instruction is to clean the pin sites daily with sterile saline solution and assess for signs of infection, such as redness, swelling, drainage, or increased pain. The patient should also be instructed to report any signs of infection to their healthcare provider immediately.
-
Question 30 of 30
30. Question
A 50-year-old male is 1-day post-operative from an anterior cervical discectomy and fusion (ACDF). The nurse auscultates stridor upon assessment. Which of the following is the MOST appropriate initial nursing intervention?
Correct
The scenario presents a patient who is 1-day post-operative from an anterior cervical discectomy and fusion (ACDF). ACDF is a surgical procedure performed to relieve pressure on the spinal cord or nerve roots in the neck. Potential complications following ACDF include airway compromise due to swelling, hematoma formation, or nerve damage. Stridor is a high-pitched, noisy breathing sound that indicates airway obstruction. In a post-operative ACDF patient, stridor is a sign of potential airway compromise and requires immediate intervention. The nurse should immediately notify the surgeon and prepare for potential intubation or other airway management interventions. While assessing the surgical site, administering pain medication, and encouraging coughing and deep breathing are important nursing interventions, they are not the priority in the presence of stridor. Airway compromise is a life-threatening complication that requires immediate attention.
Incorrect
The scenario presents a patient who is 1-day post-operative from an anterior cervical discectomy and fusion (ACDF). ACDF is a surgical procedure performed to relieve pressure on the spinal cord or nerve roots in the neck. Potential complications following ACDF include airway compromise due to swelling, hematoma formation, or nerve damage. Stridor is a high-pitched, noisy breathing sound that indicates airway obstruction. In a post-operative ACDF patient, stridor is a sign of potential airway compromise and requires immediate intervention. The nurse should immediately notify the surgeon and prepare for potential intubation or other airway management interventions. While assessing the surgical site, administering pain medication, and encouraging coughing and deep breathing are important nursing interventions, they are not the priority in the presence of stridor. Airway compromise is a life-threatening complication that requires immediate attention.